Difference between revisions of "1985 AJHSME Problem 24"

(Created page with "== Problem == In a magic triangle, each of the six whole numbers <math>10-15</math> is placed in one of the circles so that the sum, <math>S</math>, of the three numbers on ea...")
 
(Solution)
Line 20: Line 20:
  
 
== Solution ==
 
== Solution ==
To maximize <math>S,</math> we must place the largest numbers at the corners where they will be counted twice. We can write the following equation:
+
To maximize <math>S,</math> we must place the largest numbers a following equation:
 
<cmath>S = \frac{2(13+14+15) + 10+11+12}{6} = \boxed{39}.</cmath>
 
<cmath>S = \frac{2(13+14+15) + 10+11+12}{6} = \boxed{39}.</cmath>
 
The answer is D.
 
The answer is D.

Revision as of 12:38, 6 March 2021

Problem

In a magic triangle, each of the six whole numbers $10-15$ is placed in one of the circles so that the sum, $S$, of the three numbers on each side of the triangle is the same. The largest possible value for $S$ is

[asy] draw(circle((0,0),1)); draw(dir(60)--6*dir(60)); draw(circle(7*dir(60),1)); draw(8*dir(60)--13*dir(60)); draw(circle(14*dir(60),1)); draw((1,0)--(6,0)); draw(circle((7,0),1)); draw((8,0)--(13,0)); draw(circle((14,0),1)); draw(circle((10.5,6.0621778264910705273460621952706),1)); draw((13.5,0.86602540378443864676372317075294)--(11,5.1961524227066318805823390245176)); draw((10,6.9282032302755091741097853660235)--(7.5,11.258330249197702407928401219788)); [/asy]

$\text{(A)}\ 36 \qquad \text{(B)}\ 37 \qquad \text{(C)}\ 38 \qquad \text{(D)}\ 39 \qquad \text{(E)}\ 40$

Solution

To maximize $S,$ we must place the largest numbers a following equation: \[S = \frac{2(13+14+15) + 10+11+12}{6} = \boxed{39}.\] The answer is D.